A good strengthen question...

This topic has expert replies
User avatar
Legendary Member
Posts: 934
Joined: Tue Nov 09, 2010 5:16 am
Location: AAMCHI MUMBAI LOCAL
Thanked: 63 times
Followed by:14 members

A good strengthen question...

by [email protected] » Fri Jun 15, 2012 9:50 pm
Analyst: The pace of technological development brings a constant stream of new devices to the market, and many of them enjoy commercial success. But announcing new technology too soon after the introduction of a successful device can backfire. Once consumers hear about the new device, they may stop buying the one currently on sale. So, if a company wishes to announce the upcoming sale of a new device, it should wait until purchases of the old device have begun to decline.

Which of the following, if true, would best support the analyst's main assertion?

A] New technology often becomes less expensive after an initial surge in sales.

B] Media outlets, such as television programs and magazines, often report on the planned introduction of new devices while the sales of old devices are still strong.

C] Many consumers are unable to determine whether new technology is superior to current technology.

D] Surveys have shown that some consumers make only one or two technology purchases per year, whereas others make more frequent purchases.

E] Consumers tend to be loyal to technology companies whose products they enjoy using.



The correct answer is B.


I marked the answer as C. I do not know why I still feel that the correct answer is C. If the consumers are unaware to differentiate between the old technology and the new technology, then they might go for the old technology or they might go for the new technology. But the premise in the stimulus clearly states that the consumer will go for the new technology. Hence C also strengthens the argument. According to me the C also a good choice.

Please solve my query...
IT IS TIME TO BEAT THE GMAT

LEARNING, APPLICATION AND TIMING IS THE FACT OF GMAT AND LIFE AS WELL... KEEP PLAYING!!!

Whenever you feel that my post really helped you to learn something new, please press on the 'THANK' button.

Legendary Member
Posts: 784
Joined: Sun Apr 03, 2011 3:51 am
Thanked: 114 times
Followed by:12 members

by patanjali.purpose » Sat Jun 16, 2012 12:18 am
[email protected] wrote:Analyst: The pace of technological development brings a constant stream of new devices to the market, and many of them enjoy commercial success. But announcing new technology too soon after the introduction of a successful device can backfire. Once consumers hear about the new device, they may stop buying the one currently on sale. So, if a company wishes to announce the upcoming sale of a new device, it should wait until purchases of the old device have begun to decline.

Which of the following, if true, would best support the analyst's main assertion?

C] Many consumers are unable to determine whether new technology is superior to current technology.

I marked the answer as C. I do not know why I still feel that the correct answer is C. If the consumers are unaware to differentiate between the old technology and the new technology, then they might go for the old technology or they might go for the new technology. But the premise in the stimulus clearly states that the consumer will go for the new technology. Hence C also strengthens the argument. According to me the C also a good choice. Please solve my query...
We have to pick up a choice that STRENGTHEN the conclusion (under all circumstances - there should be an iota of doubt)

In C - consumers are unable to determine whether NEW is better than OLD, then two scenario can happen:
a) consumer MAY WAIT TO purchase NEW PRODUCT, or
b) consumer MAY purchase EXISTING (OLD) PRODUCT

If (a) is TRUE, THEN argument is WEAKENED, but if (b) is TRUE, THEN argument is STRENGTHENED.

Therefore, C can go either way. Drop it

User avatar
GMAT Instructor
Posts: 1248
Joined: Thu Mar 29, 2012 2:57 pm
Location: Everywhere
Thanked: 503 times
Followed by:192 members
GMAT Score:780

by Bill@VeritasPrep » Sat Jun 16, 2012 2:02 pm
Ah, the Android approach, where your smartphone is obsolete before you've opened the box. :D

I think patanjali is correct about why C doesn't quite work. If consumers can't differentiate between the new and old technology, then it is entirely possible that they will decide that the current offering is "good enough" and continue to purchase it.
Join Veritas Prep's 2010 Instructor of the Year, Matt Douglas for GMATT Mondays

Visit the Veritas Prep Blog

Try the FREE Veritas Prep Practice Test

Newbie | Next Rank: 10 Posts
Posts: 9
Joined: Sun Apr 10, 2011 6:53 am

by nh_cryptic » Sat Jun 16, 2012 11:19 pm
whats wrong with A

User avatar
Junior | Next Rank: 30 Posts
Posts: 10
Joined: Tue Nov 23, 2010 5:10 am
Location: Bangalore, India
Thanked: 1 times

by gh_chandra2000 » Mon Jun 18, 2012 3:50 am
@nh_cryptic: the question is about the sales and not about the costs..

User avatar
GMAT Instructor
Posts: 1248
Joined: Thu Mar 29, 2012 2:57 pm
Location: Everywhere
Thanked: 503 times
Followed by:192 members
GMAT Score:780

by Bill@VeritasPrep » Mon Jun 18, 2012 7:27 am
nh_cryptic wrote:whats wrong with A
The argument is entirely about sales of the old and new devices. We have no information about how changes in pricing would affect sales, so we can rule out A for being out of the scope of the argument.
Join Veritas Prep's 2010 Instructor of the Year, Matt Douglas for GMATT Mondays

Visit the Veritas Prep Blog

Try the FREE Veritas Prep Practice Test